Neoplasia MCQ

¡Supera tus tareas y exámenes ahora con Quizwiz!

*The answer is D.* The figure shows the appearance of multiple variably sized tan metastatic lesions in the liver from hematogenous spread of carcinoma. Adenocarcinomas from abdominal primary sites such as colon, pancreas, and stomach are most likely. Thromboembolism suggests a hypercoagulable state such as a paraneoplastic syndrome. Angiosarcomas of the liver are uncommon. A primary malignancy typically appears as a dominant mass, not multiple masses. Although some benign tumors, such as leiomyomas of the uterus, can be multiple, this is not the rule in the liver, and hepatic adenomas are rare. Although hepatocellular carcinomas can have "satellite" nodules, widespread nodules such as those seen in the figure are more characteristic of metastases. Leukemic infiltrates typically do not produce large mass lesions, though some lymphomas may do so. Resection of multiple metastases is usually futile.

69-year-old woman has experienced increasing malaise and a 10-kg weight loss over the past year. She dies of massive pulmonary thromboembolism. The gross appearance of the liver at autopsy is shown in the figure. Which of the following best describes the lesions seen in her liver? A Invasive angiosarcoma B Hepatocellular carcinoma C Leukemic infiltration D Metastatic adenocarcinoma E Multifocal hepatic adenomas

*The answer is C.* These children have an autosomal recessive condition known as xeroderma pigmentosum (XP). Affected individuals have extreme photosensitivity, with a 2000-fold increase in the risk of skin cancers. The DNA damage is initiated by exposure to ultraviolet light; however, nucleotide excision repair cannot occur normally in XP because inheritance of one of several XP genes is abnormal. The inheritance pattern appears to be autosomal recessive, because new mutations are unlikely to occur. Inactivation of the p53 tumor suppressor gene is found in many sporadic human cancers and in some familial cancers, but these cancers are not limited to the skin. Chromosomal translocations are often involved in the development of hematologic malignancies, although they are not often seen in skin cancers. Aspergillus flavus, found on moldy peanuts and other foods, produces the potent hepatic carcinogen aflatoxin B1. Human papillomavirus (HPV) is a sexually transmitted disease that is associated with the development of genital squamous cell carcinomas.

A 12-year-old girl and a 14-year-old boy have developed skin nodules in predominantly sun-exposed areas of their skin over the past 5 years, but their six siblings have not. On physical examination, both children are of appropriate height and weight. The skin lesions are 1- to 3-cm maculopapular nodules that are erythematous to brown-colored and have areas of ulceration. Microscopic analysis of biopsy specimens of the skin lesions shows squamous cell carcinoma. The children have no history of recurrent infections, and their parents and other relatives are unaffected. Which of the following mechanisms is most likely to produce neoplasia in these children? A Inherited mutation of the p53 gene B Chromosomal translocation C Failure of nucleotide excision repair of DNA D Ingestion of food contaminated with Aspergillus flavus E Infection with human papillomavirus

*The answer is D.* Retinoblastomas are malignant ocular tumors of young children. In cases of hereditary retinoblastoma, an affected child inherits one defective Rb allele together with one normal gene. This heterozygous state is not associated with any observable changes in the retina because 50% of the Rb gene product is sufficient to prevent the development of retinoblastoma. However, if the remaining normal Rb allele is inactivated by deletion or mutation, the loss of its suppressor function leads to the appearance of a neoplasm. This genetic process is referred to as loss of heterozygosity. The other choices have not been associated with the loss of tumor suppressor genes in somatic cells. *Diagnosis:* Retinoblastoma

A 2-year-old boy is found to have bilateral retinal tumors. Molecular studies demonstrate a germline mutation in one allele of the Rb gene. Which of the following genetic events best explains the mechanism of carcinogenesis in this patient? (A) Balanced translocation (B) Expansion of trinucleotide repeat (C) Gene amplification (D) Loss of heterozygosity (E) Maternal nondisjunction

*The answer is E.* Teratomas are benign tumors composed of tissues derived from all three primary germ layers: ectoderm, mesoderm, and endoderm. They are most common in the ovary but also occur in the testis and extragonadal sites. Teratocarcinomas (choice D) are malignant tumors that harbor embryonal carcinoma stem cells. Adenoma (choice A) is a benign tumor of epithelial origin. Chondroma (choice B) is a benign cartilaginous tumor. Hamartoma (choice C) is disorganized normal tissue. *Diagnosis:* Mature teratoma

A 20-year-old woman has an ovarian tumor removed. The surgical specimen is 10 cm in diameter and cystic. The cystic cavity is found to contain black hair and sebaceous material. Histologic examination of the cyst wall reveals a variety of benign differentiated tissues, including skin, cartilage, brain, and mucinous glandular epithelium. What is the diagnosis? (A) Adenoma (B) Chondroma (C) Hamartoma (D) Teratocarcinoma (E) Teratoma

*The answer is B.* The small, discrete nature of this mass and its slow growth with nearly unchanged size suggest a benign neoplasm. The red color is consistent with vascularity. A hemangioma is a common benign lesion of the skin. Fibroadenomas arise in the breast. Leiomyomas, which are white, arise from smooth muscle and are most common in the uterus. Lipomas are yellow fatty tumors that can occur beneath the epidermis. Melanomas are malignant and tend to increase in size quickly; many are darkly pigmented. The benign counterpart to the melanoma is the nevus, which

A 23-year-old woman has noted a nodule on the skin of her upper chest. She reports that the nodule has been present for many years and has not changed in size. It is excised, and the microscopic appearance is shown in the figure. Which of the following neoplasms is this lesion most likely to be? A Fibroadenoma B Hemangioma C Leiomyoma D Lipoma E Melanoma F Nevus

*The answer is E.* Somatic cells do not normally express telomerase, which is an enzyme that adds repetitive sequences to maintain the length of the telomere. Thus, with each round of somatic cell replication, the telomere shortens. The length of telomeres may act as a "molecular clock" and govern the lifespan of replicating cells. Because cancer cells and embryonic cells express high levels of telomerase, the reactivation of this enzyme may be important for maintaining stem cell proliferation. Most human cancers show activation of the gene for the catalytic subunit of telomerase: human telomerase reverse transcriptase. P selectin (choice D) is a cell adhesion molecule that mediates the margination of neutrophils during acute inflammation. The other choices are not involved in malignant transformation. *Diagnosis:* Embryonal carcinoma

A 25-year-old man presents 1 week after discovering that his left testicle is twice the normal size. Physical examination reveals a nontender, testicular mass that cannot be transilluminated. Serum levels of alpha-fetoprotein and human chorionic gonadotropin are normal. A hemiorchiectomy is performed, and histologic examination of the surgical specimen shows embryonal carcinoma. Compared to normal adult somatic cells, this germ cell neoplasm would most likely show high levels of expression of which of the following proteins? (A) Desmin (B) Dystrophin (C) Cytochrome c (D) P selectin (E) Telomerase

*The answer is B.* Most carcinomas begin as localized growths confined to the epithelium in which they arise. As long as these early cancers do not penetrate the basement membrane on which the epithelium rests, such tumors are labeled carcinoma in situ. When the in situ tumor acquires invasive potential and extends directly through the underlying basement membrane, it is in a position to compromise neighboring tissues and metastasize. Carcinomatosis (choice C) is a clinical term used to describe widespread dissemination of cancer. Koilocytosis (choice E) implies the presence of squamous cells with perinuclear halos and nuclear changes. It is indicative of human papillomavirus infection and carries an increased risk of carcinoma. Atypical and complex hyperplasia (choices A and D) refer to proliferative lesions of the glands within the uterine endometrium. *Diagnosis:* Cervical carcinoma, carcinoma in situ

A 25-year-old woman presents for a gynecologic examination. The cervical Pap smear shows "koilocytic atypia" characterized by perinuclear halos and wrinkled nuclei (shown in the image). A cervical biopsy reveals invasive squamous cell carcinoma. Molecular tests for human papillomavirus (HPV) in the tumor cells are positive. The patient undergoes a hysterectomy. In addition to a focus of invasive carcinoma, the pathologist identifies dysplastic squamous cells occupying the entire thickness of the cervical epithelium, with no evidence of epithelial maturation. The basal membrane in these areas appears intact. Which of the following terms best describes this cervical lesion? (A) Atypical hyperplasia (B) Carcinoma in situ (C) Carcinomatosis (D) Complex hyperplasia (E) Koilocytic atypia

*The answer is D.* Unlike RNA tumor viruses, whose oncogenes have normal cellular counterparts, the transforming genes of DNA viruses are not homologous with any cellular genes. This conundrum was resolved with the discovery that the gene products of oncogenic DNA viruses inactivate tumor suppressor proteins. For example, proteins encoded by the E6 and E7 genes of HPV16 bind p53 and pRb. The other choices are involved in the pathogenesis of neoplasia, but they are not specific for HPV. *Diagnosis:* Cervical intraepithelial neoplasia, HPV infection

A 25-year-old woman presents for a gynecologic examination. The cervical Pap smear shows "koilocytic atypia" characterized by perinuclear halos and wrinkled nuclei (shown in the image). A cervical biopsy reveals invasive squamous cell carcinoma. Molecular tests for human papillomavirus (HPV) in the tumor cells are positive. Which of the following mechanisms of disease best explains the role of HPV in the pathogenesis of neoplasia in this patient? (A) Activation of cellular oncogenes (B) Enhanced transcription of telomerase gene (C) Episomal viral replication (D) Inactivation of tumor suppressor proteins (E) Insertional mutagenesis

*The answer is A.*The patient has a classic history of familial adenomatous polyposis with numerous adenomatous polyps and malignant transformation. The earliest event in the APC → adenocarcinoma sequence is loss of APC gene function. This prevents the destruction of β-catenin in the cytoplasm, which translocates to the nucleus and coactivates transcription of several genes. The APC → β-catenin sequence is a component of the WNT signaling pathway. RAS activation occurs after the malignant transformation sequence is initiated by the APC (gatekeeper) gene. Loss of cell cycle G1 arrest occurs with p53 loss late in the sequence. Mutations in mismatch repair genes give rise to hereditary nonpolyposis colon cancer syndrome from loss of ability to repair DNA damage. The BCL2 gene is not involved in the transition from adenoma to carcinoma.

A 26-year-old man with a family history of colon carcinoma undergoes a surveillance colonoscopy. It reveals hundreds of polyps in the colon, and two focal 0.5-cm ulcerated areas. A biopsy specimen from an ulcer reveals irregularly shaped glands that have penetrated into the muscular layer. Which of the following molecular events is believed to occur very early in the evolution of his colonic disease process? A Activation of the WNT signaling pathway B Inability to hydrolyze GTP-bound RAS C Loss of heterozygosity affecting the p53 gene D Mutations in mismatch repair genes. E Translocation of BCL2 from mitochondria to cytoplasm

*The answer is B.* Approximately 5% to 10% of breast cancers are familial, and 80% of these cases result from mutations in the BRCA1 and BRCA2 genes. Onset of these familial cancers occurs earlier in life than the sporadic cancers. The protein products of these genes are involved in DNA repair. BCL2 is overexpressed in some lymphoid neoplasms. The epithelial growth factor receptor ERBB2 (HER2) overexpression is present in some sporadic breast cancers; other EGF alterations can be seen in lung, bladder, gastrointestinal, ovarian, and brain neoplasms. The HST1 mutation is seen in some gastric cancers. IL2 overexpression is associated with some T cell neoplasms. K-RAS overexpression is seen in many cancers, including some breast cancers, but the early age of onset and family history in this case strongly suggest BRCA mutations.

A 26-year-old woman has a lump in her left breast. On physical examination, she has an irregular, firm, 2-cm mass in the upper inner quadrant of the breast. No axillary adenopathy is noted. A fine-needle aspirate of the mass shows anaplastic ductal cells. The patient's 30-year old sister was recently diagnosed with ovarian cancer, and 3 years ago her maternal aunt was diagnosed with ductal carcinoma of the breast and had a mastectomy. Mutation involving which of the following genes is most likely present in this family? A BCL2 (anti-apoptosis gene) B BRCA1 (DNA repair gene) D ERBB2 (growth factor receptor gene) E HST1 (fibroblast growth factor gene) F IL2 (growth factor gene) G K-RAS (GTP-binding protein gene)

*The answer is C.* The surgical specimen reveals thousands of small adenomatous polyps on the mucosal surface of the colon. Patients with adenomatous polyposis coli have mutations in the APC tumor suppressor gene. Most cases are familial, but 30% to 50% represent new mutations. The mean age for occurrence of symptoms is 36 years. Without the APC protooncogene, cells are unable to downregulate signals from E-cadherin to b-catenin to nuclear transcription factors (myc and cyclin D) that regulate cell cycle progression. Autophagy (choice B) is a normal catabolic process in which cellular components and organelles are degraded in lysosomes. Autophagy is often a response to cell injury. It is also believed to protect cells from intracellular pathogens and slow the progression of various chronic diseases, including cancer. *Diagnosis:* Adenomatous polyposis coli

A 28-year-old man with a familial disease affecting the gastrointestinal tract undergoes a colectomy. The surgical specimen is shown in the image. Molecular studies demonstrate a germline mutation in the APC gene. The normal product of this gene (protooncogene) primarily regulates which of the following cell behaviors? (A) Apoptosis (B) Autophagy (C) Cell cycle (D) Differentiation (E) Motility

*The answer is G.* The RB gene is a classic example of the two-hit mechanism for loss of tumor suppression. About 60% of these tumors are sporadic, whereas the rest are familial from inheritance of a mutated copy of the RB gene. Loss of the second copy in retinoblasts leads to the occurrence of retinoblastoma in childhood. The RB gene controls the G1 to S transition of the cell cycle; with loss of both copies, this important checkpoint in the cell cycle is lost. The BCR-ABL fusion gene in chronic myelogenous leukemia is an example of overexpression of a gene product producing neoplasia. The BCL2 gene is an inhibitor of apoptosis. The hMSH2 gene is present in most cases of hereditary nonpolyposis colon cancer and functions in DNA repair. Many cancers have the K-RAS gene, which acts as an oncogene. The NF1 gene product acts as a tumor suppressor; this is a component of neurofibromatosis (which usually does not involve the eye), and the neoplasms typically appear at a later age. Many cancers have the p53 tumor suppressor gene mutation, but this is not typical of childhood ocular neoplasms.

A 3-year-old child has exhibited difficulty with vision in her right eye. On physical examination, there is leukocoria of the right eye, consistent with a mass in the posterior chamber. MR imaging shows a mass that nearly fills the globe. The child undergoes enucleation of the right eye. Molecular analysis of the neoplastic cells indicates absence of both copies of a gene that contributes to control of the cell cycle. Which of the following genes has most likely undergone mutation in this neoplasm? A BCR-ABL B BCL2 C hMSH2 D K-RAS E NF1 F p53 G RB

*The answer is G.* A teratoma is a neoplasm derived from totipotential germ cells that differentiate into tissues that represent all three germ layers: ectoderm, endoderm, and mesoderm. When the elements all are well differentiated, the neoplasm is "mature" (benign). Adenocarcinomas have malignant-appearing glandular elements. Fibroadenomas have a benign glandular and stromal component; they are common in the breast. Gliomas are found in the central nervous system. Hamartomas contain a mixture of cell types common to a tissue site; the lung is one site for this uncommon lesion. A mesothelioma arises from the lining of thoracic and abdominal body cavities. A rhabdomyosarcoma comprises cells that poorly resemble striated muscle; most arise in soft tissues.

A 32-year-old woman has experienced dull pelvic pain for the past 2 months. Physical examination shows a right adnexal mass. An abdominal ultrasound scan shows a 7.5-cm cystic ovarian mass. The mass is surgically excised. The surface of the mass is smooth, and it is not adherent to surrounding pelvic structures. On gross examination, the cystic mass is filled with hair. Microscopically, squamous epithelium, tall columnar glandular epithelium, cartilage, and fibrous connective tissue are present and resemble normal tissue counterparts. Which of the following is the most likely diagnosis? A Adenocarcinoma B Fibroadenoma C Glioma D Hamartoma E Mesothelioma F Rhabdomyosarcoma G Teratoma

*The answer is C.* Breast (BR) cancer (CA) susceptibility genes (BRCA1 and BRCA2) encode tumor suppressor proteins involved in checkpoint functions related to progression of the cell cycle into S phase. BRCA1 and BRCA2 proteins also promote DNA repair by binding to RAD51, a molecule that mediates DNA double-strand repair breaks. The other choices may be abnormal in neoplasia, but they are not primarily affected by BRCA1. *Diagnosis:* Breast cancer

A 33-year-old woman discovers a lump in her left breast on self-examination. Her mother and sister both had breast cancer. A mammogram demonstrates an ill-defined density in the outer quadrant of the left breast, with microcalcifications. Needle aspiration reveals the presence of malignant, ductal epithelial cells. Genetic screening identifies a mutation in BRCA1. In addition to cell cycle control, BRCA1 protein promotes which of the following cellular functions? (A) Apoptosis (B) Cell adhesion (C) DNA repair (D) Gene transcription (E) Transmembrane signaling

*The answer is B.* CD4 is a cluster-differentiation antigen of helper T lymphocytes. HMB-45 and S-100 (choices D and E) are markers for malignant melanoma, among other tumors. Calcitonin (choice A) is a peptide hormone. Desmin (choice C) is an intermediate filament protein found in cells of mesenchymal origin. *Diagnosis:* Mycosis fungoides

A 33-year-old woman presents with a diffuse scaly skin rash of 4 weeks duration. Biopsy of lesional skin reveals a cutaneous T-cell lymphoma (mycosis fungoides). Which of the following immunohistochemical markers would be most useful for identifying malignant cells in the skin of this patient? (A) Calcitonin (B) CD4 (C) Desmin (D) HMB-45 (E) S-100

*The answer is A.* The oncogenic potential of human papillomavirus (HPV), a sexually transmissible agent, is related to products of two early viral genes—E6 and E7. E7 protein binds to RB protein to cause displacement of normally sequestered transcription factors, which nullifies tumor suppressor activity of the RB protein. E6 protein binds to and inactivates the p53 gene product. Increased epidermal growth factor receptor expression is a feature seen in many pulmonary squamous cell carcinomas, and the related ERBB2 (HER2) receptor is seen in some breast carcinomas. Epigenetic modifications include DNA methylation and histone modifications which, depending on their nature, may enhance or inhibit gene expression. Inability to repair DNA damage plays a role in some colon and skin cancers. Trapping of GTP-bound RAS protein can occur in many tumors but is not related to HPV infection.

A 33-year-old woman with multiple sexual partners has had vaginal bleeding and discharge for the past 5 days. On physical examination, she is afebrile. Pelvic examination shows an ulcerated lesion arising from the squamocolumnar junction of the uterine cervix. A cervical biopsy is performed and microscopic examination reveals an invasive tumor containing areas of squamous epithelium, with pearls of keratin. In situ hybridization shows the presence of human papillomavirus type 16 (HPV-16) DNA within the tumor cells. Which of the following molecular abnormalities in this tumor is most likely related to infection with HPV-16? A Functional inactivation of the RB protein B Increased expression of epidermal growth factor receptor C Epigenetic silencing of the RB gene D Inability to repair DNA damage E Trapping of the RAS protein in a GTP-bound state

*The answer is B.* Epithelial dysplasias, especially severe dysplasias, can be precursors of carcinomas. This is a key reason for Pap smear screening. The incidence of cervical carcinoma decreases when routine Pap smears are performed. Colposcopy with biopsy is indicated to determine the extent of the lesion for removal. Though related to human papillomavirus (HPV) infection, severe dysplasias are not amenable to antibiotic therapy. Ovarian neoplasms are not related to cervical dysplasias or carcinomas. In general, cervical cancers are not related to hereditary syndromes, and cervical dysplasias are not hereditary. Screening of family members is appropriate for those who have risk factors, such as multiple sexual partners. Regression of a severe dysplasia is unlikely.

A 34-year-old sexually active woman undergoes a routine physical examination. There are no abnormal findings. A Pap smear is obtained as part of the pelvic examination. Cytologically, the cells obtained on the smear from the cervix show severe epithelial dysplasia (high-grade squamous intraepithelial lesion). Which of the following therapeutic options is most appropriate for this woman? A Antibiotic therapy B Excision C Ovarian removal D Screening of family members E Watchful waiting

*The answer is A.* Benign tumors arising from a glandular epithelium are termed adenomas. Patients with a prolactin-secreting pituitary adenoma present with amenorrhea and galactorrhea. Ectopic islands of normal tissue are called choristomas (choice B). Localized, disordered differentiation during development results in a hamartoma (choice C). Papillomas (choice D) do not occur in the pituitary. Benign tumors that arise from germ cells and contain all three germ layers are termed teratomas (choice E). *Diagnosis:* Pituitary adenoma, prolactinoma

A 35-year-old woman complains of nipple discharge and irregular menses of 5 months duration. Physical examination reveals a milky discharge from both nipples. MRI shows an enlargement of the anterior pituitary. Which of the following is the most likely histologic diagnosis of this patient's pituitary tumor? (A) Adenoma (B) Choristoma (C) Hamartoma (D) Papilloma (E) Teratoma

*The answer is C.* The MYC oncogene is commonly activated in Burkitt lymphoma because of a t(8;14) translocation. The MYC gene binds DNA to cause transcriptional activation of growth-related genes such as that for cyclin D1, resulting in activation of the cell cycle. EGF (such as HER2 in breast cancers) encodes the epithelial growth factor receptor located on the cell surface. p53 and APC are tumor suppressor genes that are inactivated in many cancers, including colon cancer. RAS oncogene encodes a GTP-binding protein that is located under the cell membrane.

A 38-year-old woman has abdominal distention that has been worsening for the past 6 weeks. An abdominal CT scan shows bowel obstruction caused by a 6-cm mass in the jejunum. At laparotomy, a portion of the small bowel is resected. Flow cytometric analysis of a portion of the tumor shows a clonal population of B lymphocytes with high S phase. Translocation with activation of which of the following nuclear oncogenes is most likely to be present in this tumor? A APC B EGF C MYC D p53 E RAS

*The answer is C.* Although the hepatitis B virus (HBV) and hepatitis C virus (HCV) genomes do not encode for any transforming proteins, the regenerating hepatocytes are more likely to develop mutations, such as inactivation of p53. HBV does not have a consistent site of integration in the liver cell nuclei, and it does not contain viral oncogenes. Many DNA viruses, such as human papillomavirus (HPV), inactivate tumor suppressor genes, but there is no convincing evidence that HBV or HCV can bind to p53 or RB proteins. Also, the HBV-encoded regulatory element, called HBx, disrupts normal growth of infected hepatocytes. Neither HBV nor HCV infects immune cells.

A 40-year-old man has a history of intravenous drug use. Physical examination shows needle tracks in his left antecubital fossa. He has mild scleral icterus. Serologic studies for HBsAg and anti-HCV are positive. He develops hepatocellular carcinoma 15 years later. Which of the following viral characteristics best explains why this patient developed hepatocellular carcinoma? A Viral integration in the vicinity of proto-oncogenes B Viral capture of proto-oncogenes from host cellular DNA C Viral inflammatory changes with genomic damage D Viral inactivation of RB and p53 gene expression E Viral infection of inflammatory cells with host immunosuppression

*The answer is A.* A discrete small mass such as that described is probably benign. Adenomas arise from epithelial surfaces. Though adenocarcinoma may arise from a colonic adenoma, such malignant lesions tend to be larger and more irregular. A choristoma is a benign mass composed of tissues not found at the site of origin. A hamartoma is a rare benign mass composed of tissues usually found at the site of origin. A hyperplastic colonic lesion tends to be smaller and flatter. A sarcoma is a malignant neoplasm arising in mesenchymal tissues, not in epithelium.

A 40-year-old man has a positive stool guaiac test during a routine physical examination. A colonoscopy is performed and a 0.9-cm, circumscribed, pedunculated mass on a short stalk is found in the upper rectum. Which of the following terms best describes this lesion? A Adenoma B Carcinoma C Choristoma D Hamartoma E Hyperplasia F Sarcoma

*The answer is A.* The BCL2 gene controls production of a protein that inhibits apoptosis, and overexpression of this gene allows accumulation of abnormal cells in lymphoid tissues. Gene amplifications typically affect the ERBB2 (HER2) and MYC oncogenes. Increased tyrosine kinase activity results from mutations affecting the ABL oncogene. Loss of cell cycle inhibition results from loss of tumor suppressor genes such as p53. Reduced DNA repair occurs in the inherited disorder xeroderma pigmentosum.

A 40-year-old man notices an increasing number of lumps in his groin and armpit over the past 5 months. On physical examination, he has generalized nontender lymph node enlargement and hepatosplenomegaly. An inguinal lymph node biopsy specimen shows a malignant tumor of small, well-differentiated lymphoid cells. Immunostaining of the tumor cells with antibody to BCL2 is positive in the lymphocytic cell nuclei. Which of the following mechanisms has most likely produced this lymphoid neoplasm? A Diminished apoptosis B Gene amplifications C Increased tyrosine kinase activity D Loss of cell cycle inhibition E Reduced DNA repair

*The answer is C.* The highest incidence of stomach cancer occurs in Japan, where the disease is almost ten times as frequent as it is among American whites. A study of Japanese residents of Hawaii found that emigrants from Japanese regions with the highest risk of stomach cancer continued to exhibit an excess risk in Hawaii. By contrast, their offspring who were born in Hawaii had the same incidence of this cancer as American whites. The highest incidence of colorectal cancer is found in the United States (choice E). *Diagnosis:* Gastric cancer

A 42-year-old man presents with upper gastrointestinal bleeding. Upper endoscopy and biopsy reveal gastric adenocarcinoma. Which country of the world has the highest incidence of this malignant neoplasm? (A) Argentina (B) Canada (C) Japan (D) Mexico (E) United States

*The answer is C.* Lymphatic spread, especially to regional lymph nodes draining from the primary site, is typical of a carcinoma. An intraductal carcinoma has not extended beyond the basement membrane, but an infiltrating carcinoma has acquired the ability to invade and spread via metastasis. The primary site may be difficult to detect if small or deep, and hence the need for radiologic imaging, such as mammography. A fibroadenoma is a benign neoplasm and cannot invade or metastasize. Infection from a breast abscess can spread to the lymph nodes, but the resulting nodal enlargement is typically associated with pain—a cardinal sign of acute inflammation. Sarcomas uncommonly metastasize to lymph nodes, and a leiomyosarcoma of breast is rare.

A 44-year-old woman feels painless lumps in her armpit, which were not present a month ago. On examination, right axillary lymphadenopathy is present. The nodes are painless but firm. Which of the following is the most likely lesion in her right breast? A Acute mastitis with abscess B Fibroadenoma C Infiltrating lobular carcinoma D Intraductal carcinoma E Leiomyosarcoma

*The answer is A.* Kaposi sarcoma is the most common neoplasm associated with acquired immunodeficiency syndrome (AIDS). The neoplastic cells contain sequences of a novel virus, HHV-8, which is also known as Kaposi sarcoma-associated herpesvirus. In addition to infecting the spindle cells of Kaposi sarcoma, HHV-8 is lymphotropic and has been implicated in two uncommon B-cell lymphoid malignancies, namely, primary effusion lymphoma and multicentric Castleman disease. Like other DNA viruses, the HHV-8 genome encodes proteins that interfere with the p53 and pRb tumor suppressor pathways. The other choices are hereditary conditions associated with cancer; however, these patients do not typically acquire Kaposi sarcoma. The predominant malignancy seen in patients with ataxia telangiectasia (choice B) is lymphoma/ leukemia. *Diagnosis:* Kaposi sarcoma, AIDS

A 45-year-old man presents with a 9-month history of a reddish nodule on his foot. Biopsy of the nodule discloses a poorly demarcated lesion composed of fibroblasts and endothelial-like cells lining vascular spaces. Further work-up identifies similar lesions in the lymph nodes and liver. The tumor cells contain sequences of human herpesvirus-8 (HHV-8). This patient most likely has which of the following diseases? (A) Acquired immunodeficiency (B) Ataxia telangiectasia (C) Li-Fraumeni syndrome (D) Neurofi bromatosis type I (E) Xeroderma pigmentosum

*The answer is B.* Leiomyoma is the most common benign tumor of the uterus, usually arising in women of reproductive age. It originates from smooth muscle cells of the myometrium. None of the other choices are benign tumors of smooth muscle. *Diagnosis:* Leiomyoma of uterus

A 45-year-old woman presents with abdominal pain and vaginal bleeding. A hysterectomy is performed and shows a benign tumor of the uterus derived from a smooth muscle cell. What is the appropriate diagnosis? (A) Angiomyolipoma (B) Leiomyoma (C) Leiomyosarcoma (D) Myxoma (E) Rhabdomyoma

*The answer is C.* The cellular and molecular mechanisms of hyperplasia are related to the control of cell proliferation and provide a basis for further genetic changes that can lead to neoplasia. Endometrial hyperplasia refers to a spectrum that ranges from simple glandular crowding to conspicuous proliferation of atypical glands. These changes are often difficult to distinguish from carcinoma. The risk of developing endometrial cancer increases with higher degrees of endometrial hyperplasia. Estrogen exposure is thought to be a risk factor for both endometrial hyperplasia and endometrial carcinoma. Neoplastic transformation may occur in the setting of a metaplastic epithelium (e.g., cancers of the lung, cervix, stomach, and bladder); however, metaplasia (choice E) does not precede the development of uterine adenocarcinoma. The other choices do not represent risk factors for cancer. *Diagnosis:* Endometrial adenocarcinoma

A 48-year-old nulliparous woman complains that her menstrual blood flow is more abundant than usual. An ultrasound examination reveals a polypoid mass in the uterine fundus. The patient subsequently, undergoes a hysterectomy, which reveals a poorly differentiated endometrial adenocarcinoma. The development of this neoplasm was preceded by which of the following histopathologic changes in the glandular epithelium? (A) Atrophy (B) Hydropic swelling (C) Hyperplasia (D) Hypertrophy (E) Metaplasia

*The answer is C.* Several pathologic mechanisms play a role in the development of tumor metastases. The tumor cells first must become discohesive and detach from the primary site, degrade the basement membrane and interstitial connective tissue, and then attach elsewhere to become metastases. Reduced expression of adhesion molecules such as E-cadherins promotes metastases. Tumor cells can elaborate, not reduce, proteases such as metalloproteinases to promote discohesiveness. Expression of estrogen and progesterone receptors predicts breast cancer responsiveness to antihormone therapy, and there is a monoclonal antibody, trastuzumab, that targets HER-2, a form of epidermal growth factor receptor. Tyrosine kinase receptors within cells aid in signaling cell growth.

A 48-year-old woman notices a lump in her left breast. On physical examination she has a firm, nonmovable, 2-cm mass in the upper outer quadrant of the left breast. There are enlarged, firm, nontender lymph nodes in the left axilla. A fine-needle aspiration biopsy is performed, and the cells present are consistent with carcinoma. A lumpectomy with axillary lymph node dissection is performed, and carcinoma is present in two of eight axillary nodes. Reduced expression of which of the following molecules by the tumor cells is most likely responsible for the lymph node metastases? A Estrogen receptors B ERBB2 (HER-2) C E-cadherin D Progesterone receptors E Tyrosine kinases

*The answer is B.* The small cells have scant cytoplasm but marked hyperchromatism, consistent with small cell anaplastic carcinoma. This patient has Cushing syndrome resulting from ectopic corticotropin production by the tumor, a form of paraneoplastic syndrome common to small cell carcinomas of the lung. Such small cell carcinomas are of neuroendocrine derivation. A syndrome of inappropriate antidiuretic hormone (SIADH) secretion from small cell carcinomas is also common, but leads to hyponatremia as well as edema. Erythropoietin production with polycythemia is more likely to be associated with a renal cell carcinoma. Insulin and gastrin production are most often seen in islet cell tumors of the pancreas. Hypercalcemia from a parathyroid hormone-related peptide (PTHrP) is more typically associated with pulmonary squamous cell carcinomas.

A 49-year-old man experiences an episode of hemoptysis. On physical examination, he has puffiness of the face, pedal edema, and systolic hypertension. A chest radiograph shows an irregular perihilar 5-cm mass of the right lung. Laboratory studies show normal serum electrolytes. A transbronchial biopsy is performed, and the microscopic findings are shown in the figure. A bone scan shows no metastases. Immunohistochemical staining of the tumor cells is most likely to be positive for which of the following? A Antidiuretic hormone B Corticotropin C Erythropoietin D Insulin E Parathyroid hormone-related peptide

*The answer is B.* This is an example of chromosomal translocation that brings BCL2, an anti-apoptosis gene, close to another gene (immunoglobulin heavy chain gene). The BCL2 gene becomes subject to continuous stimulation by the adjacent enhancer element of the immunoglobulin gene, leading to overexpression. The APC gene is mutated in sporadic colon cancers and cancers associated with familial polyposis coli. The BRCA1 gene mutation is seen in some breast cancers. The c-MYC gene is found on chromosome 8, and the t(8;14) translocation seen in many Burkitt lymphomas leads to MYC overexpression. The IL2 mutation may be present in some T cell neoplasms. K-RAS mutations are present in many cancers, but not typically lymphoid malignancies.

A 49-year-old man has a lump near his right shoulder that has been increasing in size for the past 8 months. On physical examination, a 4-cm, firm, nontender mass is palpable in the right supraclavicular region. The mass is excised, and microscopically it shows a lymphoid neoplasm. Karyotypic analysis of the cells shows a chromosomal translocation, t(14;18), bringing the immunoglobulin heavy chain gene together with another gene. Which of the following genes is most likely activated by this translocation? A APC (tumor suppressor gene) B BCL2 (anti-apoptosis gene) C BRCA1 (DNA repair gene) D c-MYC (transcription factor gene) E IL2 (growth factor gene) F K-RAS (GTP-binding protein gene)

*The answer is D.* This patient has a classic history and t(9;22) translocation with chronic myelogenous leukemia. The translocation causes uncontrolled nonreceptor tyrosine kinase activity of the BCR-ABL fusion gene. These patients undergo remission with drugs such as imatinib that inhibit tyrosine kinases. Agents that activate caspases theoretically may help in many cases, especially when apoptosis is blocked as in tumors with BCL2 overexpression. Antibodies to epithelial growth factor receptors, such as ERBB2 (HER2) receptors, are beneficial in certain breast tumors with amplification of this gene. Delivery of p53 into cells by viral vectors has not yet been proven to be valuable in cancer treatment, and it is not used in chronic myelogenous leukemia. Translocation of β-catenin to the nucleus occurs in colon cancers when there is mutational loss of APC genes.

A 50-year-old woman has had easy fatigability and noted a dragging sensation in her abdomen for the past 5 months. Physical examination reveals that she is afebrile. She has marked splenomegaly, but no lymphadenopathy. Laboratory studies show her total WBC count is 250,000/mm3 with WBC differential count showing 64% segmented neutrophils, 11% band neutrophils, 7% metamyelocytes, 5% myelocytes, 4% myeloblasts, 3% lymphocytes, 2% basophils, 2% eosinophils, and 2% monocytes. A bone marrow biopsy is performed, and karyotypic analysis of the cells reveals a t(9;22) translocation. Medical treatment with a drug having which of the following modes of action is most likely to produce a complete remission in this patient? A Activating cellular caspases B Antibody binding to EGF receptors C Delivering normal p53 into cells with viral vectors D Inhibiting tyrosine kinase activity E Preventing translocation of β-catenin to the nucleus

*The answer is A.* Gene amplification of HER2 can cause this abnormal expression.

A 50-year-old woman notes a lump in her breast. A fine needle aspiration biopsy is performed, and on microscopic examination a carcinoma is present. Immunohistochemical shows strong HER2 positivity in these malignant cells. Through which of the following mechanisms has this abnormal expression most likely occurred? A. Gene amplification B. Epigenetic alteration C. Growth factor gene point mutation D. Reduced miRNA expression E. Balanced translocation

*The answer is A.* The patient shows signs and symptoms of Cushing syndrome (upper truncal obesity and hypercortisolism). The surgical specimen reveals a circumscribed tumor of the adrenal cortex that produces cortisol. Histologic examination of this tumor reveals nests of clear, lipid-laden epithelial cells. None of the other choices describe a benign tumor of glandular epithelial origin. *Diagnosis:* Adrenal adenoma, Cushing syndrome

A 50-year-old woman presents with a 2-year history of uppertruncal obesity and depression. Serum levels of glucose and cortisol are elevated. A CT scan of the abdomen reveals a 2-cmsuprarenal mass. The surgical specimen is shown in the image. If this neoplasm is benign, which of the following is the most appropriate diagnosis? (A) Adenoma (B) Chondroma (C) Lipoma (D) Papilloma (E) Teratoma

*The answer is C.* Secondary descriptors are used to refer to a tumor's morphologic and functional characteristics. Papillomatosis (choice E) describes frond-like structures. Medullary (choice D) signifies a soft cellular tumor, whereas scirrhous or desmoplastic implies dense fibrous stroma. Colloid carcinomas (choice A) secrete abundant mucus. Comedocarcinoma (choice B) is an intraductal neoplasm in which necrotic material can be expressed from the ducts. *Diagnosis:* Breast cancer

A 50-year-old woman presents with a lump in her breast. A 4-cm firm and fixed mass is noted on breast examination. Excisional biopsy reveals malignant cells that form gland-like structures and solid nests, surrounded by a dense collagenous stroma. A connective tissue stain (trichrome) of the biopsy is shown in the image. Which of the following descriptive terms best describes the blue areas observed in this specimen? (A) Colloid carcinoma (B) Comedocarcinoma (C) Desmoplastic change (D) Medullary carcinoma (E) Papillomatosis

*The answer is D.* A true neoplasm is a monoclonal proliferation of cells, whereas a reactive proliferation of cells is not monoclonal. Molecular genetic analysis, such as allelotype analysis with microsatellite markers, shows clonality. Reactive and neoplastic cellular proliferations may have similar histochemical and immunohistochemical staining patterns based on the type of cells and proteins that are present. Flow cytometry is effective at indicating the DNA content, aneuploidy, and growth fraction, but does not indicate clonality.

A 50-year-old woman undergoes screening colonoscopy as part of a routine health maintenance work-up. An isolated 1-cm pedunculated polyp is found in the sigmoid colon. The excised polyp histologically shows well-differentiated glands with no invasion of the stalk. Which of the following investigational research procedures can distinguish most clearly whether the polyp represents hyperplasia of the colonic mucosa or a tubular adenoma? A Flow cytometry to quantitate cells in the S phase B Histochemical staining for mucin C Immunohistochemical staining for keratin D Molecular marker of clonality

*The answer is F.* Vinyl chloride is a rare cause of liver cancer. This causal relationship was easy to show, however, because hepatic angiosarcoma is a rare neoplasm. Arsenic is a risk factor for skin cancer. Asbestos exposure is linked to pleural malignant mesothelioma and to bronchogenic carcinomas in smokers. Benzene exposure is linked to leukemias. Beryllium exposure can produce interstitial lung disease and lung cancer. Nickel exposure increases the risk of respiratory tract cancers. Exposure to naphthalene compounds is a risk factor for cancers of the urinary tract.

A 51-year-old man who works in a factory that produces plastic pipe has experienced weight loss, nausea, and vomiting over the past 4 months. On physical examination, he has tenderness to palpation in the right upper quadrant of the abdomen, and the liver span is increased. Laboratory findings include serum alkaline phosphatase, 405 U/L; AST, 45 U/L; ALT, 30 U/L; and total bilirubin, 0.9 mg/dL. An abdominal CT scan shows a 12-cm mass in the right lobe of the liver. A liver biopsy is performed, and microscopic examination shows a malignant neoplasm of endothelial cells. The patient has most likely been exposed to which of the following agents? A Arsenic B Asbestos C Benzene D Beryllium E Nickel F Vinyl chloride G Naphthalene

*The answer is E.* Neuroendocrine tumors may synthesize a number of hormones. The presence of small, membrane-bound granules with a dense core is a feature of these neoplasms. Dense granules are visible by electron microscopy. In this way, electron microscopy may aid in the diagnosis of poorly differentiated cancers, whose classification is problematic by light microscopy. Carcinomas often exhibit desmosomes and specialized junctional complexes, which are structures that are not typical of sarcomas or lymphomas. Myelin figures (choice D) are seen in patients with inherited lysosomal storage disease. Councilman bodies (choice A) are apoptotic hepatocytes (acidophilic bodies). *Diagnosis:* Small cell carcinoma of lung, paraneoplastic syndrome

A 52-year-old woman presents with a 1-year history of upper truncal obesity and moderate depression. Physical examination shows hirsutism and moon facies. A CT scan of the thorax displays a hilar mass. A transbronchial lung biopsy discloses small cell carcinoma. Electron microscopy of this patient's lung tumor will most likely reveal which of the following cytologic features? (A) Councilman bodies (B) Hyperplasia of endoplasmic reticulum (C) Mitochondrial calcification (D) Myelin figures in lysosomes (E) Neuroendocrine granules

*The answer is A.* The PET scan is based upon selective uptake of a glucose derivative into tumor cells. The Warburg effect occurs when cancer cells shift their metabolism to aerobic glycolysis for selective growth advantage under harsh circumstances. Glycolysis also yields pyruvate for anabolic demands of increased tumor doublings. The p53 and c-MYC genes favor this metabolic change, whereas PTEN inhibits tumor cell autophagy, giving cancer cells an edge in growth. Cancer cells are less differentiated than normal cells and thus have decreased ability to do many complex biochemical processes, so they favor a simple one—glycolysis. Gluconeogenesis is a function of hepatocytes in response to decreased caloric intake. The HMP shunt and Krebs cycle are more useful to normal cells maintaining themselves at the status quo. Neoplasms generate large amounts of purines from cell divisions and cell turnover that must be eliminated as uric acid, but neoplastic cells do not perform this task.

A 53-year-old man diagnosed with oral cancer and treated with radiation and chemotherapy 1 year ago now has a positron emission tomography (PET) scan of his neck that shows a single focus of increased uptake. This focus is resected and microscopic examination shows that it is a metastasis. Molecular analysis of this cancer shows p53, PTEN, and c-MYC gene mutations. Which of the following metabolic pathways is most likely up-regulated to promote his cancer cell survival and proliferation? A Aerobic glycolysis B Gluconeogenesis C Hexose monophosphate shunt D Oxidative phosphorylation E Purine degradation

*The answer is D.* A malignant epithelial neoplasm arises in the mucosa but has a tendency to invade locally. A benign neoplasm is often well circumscribed, and compressed normal surrounding tissue appears to form a discrete border. This localized lesion can be resected easily, with adequate margins. Without evidence for spread outside the colon, chemotherapy is unlikely to be of benefit. The biopsy specimen shows a malignant lesion; it must be removed before it increases in size and invades locally or metastasizes. If there is no family history, a familial cancer with high risk of recurrence from multiple polyps is unlikely; local excision is adequate. Such a solitary mucosal lesion is unlikely to represent a metastasis.

A 53-year-old woman has noticed increasing malaise. On physical examination, there are no abnormal findings, but a stool guaiac test is positive. Her hemoglobin level is 7.9 g/dL. A colonoscopy is performed, and a 3-cm sessile mass is found in the cecum. A biopsy specimen of the mass shows a moderately differentiated adenocarcinoma confined to the mucosa. An abdominal CT scan shows no lymphadenopathy or hepatic lesions. Given this information, which of the following is the best course of action? A Administer a multiagent chemotherapeutic regimen B Observe the lesion for further increase in size C Remove the entire colon to prevent a recurrence D Resect the tumor and some normal surrounding tissue E Search for a primary malignancy in another organ

*The answer is C.* This patient's lung shows numerous nodules of metastatic carcinoma corresponding to "cannon ball" metastases seen radiologically. Pulmonary metastases are more common than primary lung tumors, and the histologic appearance of most metastases resembles that of the primary tumor. Persons with ulcerative colitis (such as this patient) have a higher risk of colorectal cancer than the general population. The risk is related to the extent of colorectal involvement and the duration of the inflammatory disease. Carcinoid tumor of the lung (choice A) and primary lung cancer (choice B) would not typically show multiple, circumscribed nodules. Miliary tuberculosis (choice D) and sarcoidosis (choice E) feature mm-sized inflammatory nodules (minute granulomas). *Diagnosis:* Metastatic cancer, metastatic carcinoma of the lung

A 53-year-old woman with a longstanding history of ulcerative colitis presents with increasing chest pain and shortness of breath of 2 months duration. She reports four recent episodes of hemoptysis. The patient subsequently develops overwhelming sepsis and expires. A section through the right lung is examined at autopsy (shown in the image). What is the appropriate diagnosis? (A) Carcinoid tumor of the lung (B) Primary adenocarcinoma of the lung (C) Metastatic carcinoma of the lung (D) Miliary tuberculosis (E) Sarcoidosis

*The answer is A.* Infiltrating ductal and intraductal carcinoma are present in the figure. Increased expression of ERBB2 (HER2) can be detected immunohistochemically and by fluorescence in situ hybridization (FISH) in the biopsy specimen. One third of breast cancers may show this change. Such amplification is associated with a poorer prognosis. Detection of a specific gene product in the tissue has value for determination of treatment and prognosis. BRCA1 and p53 mutations, if inherited in the germ line, can predispose the patient to breast cancer and other tumors. With BRCA1, there is family history of breast cancer, often at a young age. The tumor suppressor gene p53 mutations predispose to many types of cancers. An inherited deletion of RB gene predisposes to retinoblastoma. The BCR-ABL fusion product, seen in chronic myeloid leukemia, often results from t(9;22).

A 54-year-old woman notes a lump in her right breast. Physical examination shows a 2-cm mass fixed to the underlying issues beneath the areola and three firm, nontender, lymph nodes palpable in the right axilla. There is no family history of cancer. An excisional breast biopsy is performed, and microscopic examination shows the findings in the figure. Over the next 6 months, additional lymph nodes become enlarged, and CT scans show nodules in the lung, liver, and brain. Which of the following molecular abnormalities is most likely to be found in her carcinoma cells? A Amplification of the ERBB2 (HER2) gene B Deletion of one RB gene copy C Fusion of BCR and C-ABL genes D Inactivation of one BRCA1 gene copy E Mutation of one p53 gene copy

*The answer is C.* A p53 mutation involving both wild type alleles is one of the most common genetic alterations in human cancers, including the most common cancers—lung, colon, and breast. The loss of this tumor suppressor activity indicates that the cell cycle is not properly arrested in the late G1 phase, and when DNA damage occurs, DNA repair cannot be completed before the cell proliferates. Inability to hydrolyze GTP is a result of RAS oncogene activation. Growth factors such as EGF are activators of the cell cycle to promote cell growth. Microsatellite instability occurs with mutation in genes, such as hMSH2, that repair DNA damage. Transcriptional activation is a feature of the MYC proto-oncogene.

A 55-year-old man has had hemoptysis and worsening cough for the past month. On physical examination, wheezes are auscultated over the right lung posteriorly. A chest radiograph shows a 6-cm right perihilar mass. A fine-needle aspiration biopsy is performed and yields cells with the microscopic appearance of non-small cell bronchogenic carcinoma. Molecular analysis of the neoplastic cells shows a p53 gene mutation. Which of the following mechanisms has most likely produced the neoplastic transformation? A Inability to hydrolyze GTP B Growth factor receptor activation C Loss of cell cycle arrest D Microsatellite instability E Transcriptional activation

*The answer is B.* The photograph shows pigmented cells in the vertebral bodies of a person who died of malignant melanoma. This autopsy fi nding illustrates the point that accurate tumor identification depends on morphologic resemblance to normal tissue. Tumor emboli in this case probably reached bone after surviving passage through the pulmonary microcirculation. None of the other tumors show pigmentation. *Diagnosis:* Melanoma

A 55-year-old woman presents with increasing weight loss and fatigue and subsequently dies of metastatic cancer. The vertebral column at autopsy is shown in the image. What is the diagnosis? (A) Chondrosarcoma (B) Melanoma (C) Multiple myeloma (D) Osteosarcoma (E) Rhabdomyosarcoma

*The answer is D.* Estrogen, similar to many other hormones and drugs, by itself is not carcinogenic, but it is responsible for stimulation of endometrial growth (hyperplasia), which has a promoting effect when cellular mutations occur to produce carcinoma. Inherited susceptibility can never be completely excluded when an individual has two tumors; this can occur in patients with inherited mutations in the p53 gene. In this case, however, there is a clear hormonal basis for the second tumor. Faulty tumor suppressor genes are not involved in hormonal promotion of a neoplasm. A paraneoplastic syndrome results from ectopic secretion of a hormone by tumor (e.g., lung cancer cells producing corticotropin). Tumor heterogeneity does not refer to two separate kinds of neoplasms; it refers to heterogeneity with a given tumor or metastasis.

A 56-year-old woman has had vaginal bleeding for 1 week. Her last menstrual period was 10 years ago. On physical examination, a lower abdominal mass is palpated. An endometrial biopsy is performed and shows endometrial carcinoma. An abdominal CT scan shows a 6-cm mass in the left ovary. A total abdominal hysterectomy is performed. Microscopically, the ovarian mass is a granulosa-theca cell tumor producing estrogen. Which of the following best describes the relationship between these two neoplasms? A Genetic susceptibility to tumorigenesis B Mutational inactivation of a tumor suppressor gene C Paraneoplastic syndrome D Promotion of carcinogenesis E Tumor heterogeneity

*The answer is A.* The cells shown in the figure show marked pleomorphism and hyperchromatism (anaplasia), and it is difficult to discern the cell of origin because no differentiation is noted. A bizarre tripolar mitotic figure is present. This degree of anaplasia is consistent with an aggressive, high-grade malignancy called anaplastic carcinoma. Apoptosis is single cell necrosis, but the cells shown appear viable and not fragmented. Dysplasia refers to changes within an epithelium that presage a neoplasm. Metaplasia with one epithelial cell type substituted for another may presage dysplasia and malignancy. Well-differentiated neoplasms tend to be less aggressive and slower growing and resemble the cell of origin.

A 57-year-old woman has experienced an increasing feeling of fullness in her neck along with a 3-kg (7-lb) weight loss over the past 3 months. On physical examination, there is a firm, fixed mass in a 3 × 5 cm area in the right side of the neck. A CT scan shows a solid infiltrating mass in the region of the right lobe of the thyroid gland. A biopsy of the mass is performed and the microscopic appearance is shown in the figure. All areas of the tumor have similar morphology. Which of the following terms best describes this neoplasm? A Anaplastic B Apoptotic C Dysplastic D Metaplastic E Well-differentiated

*The answer is A.* The cancer has spread fo the lymph nodes, but it has not metastasized.

A 58-year-old woman has a routine physical examination. A 4 cm diameter non-tender, fixed and hard mass is palpated in her right breast. Another 1 cm non-tender mass is palpable in the right axilla. In addition, a chest radiograph reveals a single 2 cm nodule in the right lung. Pathologic examination of the breast and lymph nodes shows adenocarcinoma, yet the lung nodule is a granulomatous infection. Which of the following TNM classifications best indicates the stage of her disease? A. T2 N1 M0 B T2 N0 M1 C T2 N1 M1 D T2 N0 M0

*The answer is C.* The first event in tumor cell invasion is breach of the basement membrane that separates an epithelium from the underlying mesenchyme. After invading the interstitial tissue, malignant cells penetrate lymphatic or vascular channels (choice D). In the lymph nodes, communications between the lymphatics and venous tributaries allow malignant cells access to the systemic circulation. The other choices are important for tumor metastases, but they occur later than basement membrane invasion. *Diagnosis:* Adenocarcinoma of colon

A 58-year-old woman with colon cancer presents with 3 months of increasing shortness of breath. A chest X-ray reveals numerous, bilateral, round masses in both lungs. Histologic examination of an open-lung biopsy discloses malignant gland-like structures, which are nearly identical to the colon primary. Which of the following changes in cell behavior was the first step in the process leading to tumor metastasis from the colon to the lung in this patient? (A) Arrest within the circulating blood or lymph (B) Exit from the circulation into a new tissue (C) Invasion of the underlying basement membrane (D) Penetration of vascular or lymphatic channels (E) Stimulation of angiogenesis within the pulmonary metastases

*The answer is B.* Colorectal cancer is asymptomatic in its initial stages. As the tumor grows, the most common sign is occult blood in feces, especially when the tumor is in the proximal portion of the colon. Chronic, asymptomatic bleeding typically causes iron- deficiency anemia. Adenocarcinomas of the colon usually express CEA, a glycoprotein that is released into the circulation and serves as a serologic marker for these tumors.

A 59-year-old man complains of progressive weakness. He reports that his stools are very dark. Physical examination demonstrates fullness in the right lower quadrant. Laboratory studies show iron deficiency anemia, with a serum hemoglobin level of 7.4 g/dL. Stool specimens are positive for occult blood. Colonoscopy discloses an ulcerating lesion of the cecum. Which of the following serum tumor markers is most likely to be useful for following this patient after surgery? (A) Alpha-fetoprotein (B) Carcinoembryonic antigen (C) Chorionic gonadotropin (D) Chromogranin (E) Coagulation factor VIII

*The answer is E.* Histologic sections from malignant neoplasms are frequently assessed with a panel of immunostains to detect antigenic characteristics, such as protein expression, to aid in characterizing the cell of origin, as well as provide information in selection of treatment protocols. In this case, the immunostains are likely to reveal that this neoplasm is a high-grade urothelial carcinoma. A cytology smear shows light microscopic findings helpful to screen for malignancy, but the findings often fall short of diagnosing a specific cell type. The other listed techniques are best for determination of treatment and prognosis.

A 59-year-old man has noticed blood in his urine for the past week. Cystoscopy shows a 4-cm exophytic mass involving the right bladder mucosa near the trigone. After biopsy specimens are obtained, he undergoes a radical cystectomy. Examination of the excised specimen shows an anaplastic carcinoma that has infiltrated the bladder wall. Which of the following techniques applied to the cells from his neoplasm is most likely to categorize the cell of origin? A Chromosomal karyotyping B Cytologic smear C DNA microarray D Flow cytometric analysis E Immunohistochemistry

*The answer is D.* Lung carcinoma is the cause of most cancer-related deaths in the United States and Western Europe in men and women. The second most common cause of death from cancer in women is breast cancer (choice B). One of the most common findings in patients with cancer is anemia, but the mechanism for this paraneoplastic syndrome is not clear. The anemia is usually normocytic and normochromic, although iron deficiency anemia is common in cancers that bleed into the gastrointestinal tract. *Diagnosis:* Lung cancer

A 59-year-old woman complains of "feeling light-headed" and losing 5 kg (11 lb) in the last month. A CBC reveals a normocytic, normochromic anemia. The patient subsequently dies of metastatic cancer. Based on current epidemiologic data for cancer-associated mortality in women, which of the following is the most likely primary site for this patient's malignant neoplasm? (A) Brain (B) Breast (C) Colon (D) Lung (E) Urinary bladder

*The answer is C.* Some neoplasms, including certain lymphomas, have a high proportion of cells in the replicative pool (i.e., have high growth fraction). They grow rapidly and respond rapidly to drugs that kill dividing cells. Poor vascularity would not favor rapid growth, and many neoplasms elaborate growth factors that promote vascular proliferation. Monoclonality rather than polyclonality is typical of malignant tumors, though subclones of neoplastic cells do arise over time. Aggressive neoplasms tend to be more likely to metastasize. Tumors that are highly antigenic are likely to be controlled by the immune system and not to be rapidly growing.

A 60-year-old man has noted a nodule in his neck that has increased rapidly in size over the past 2 months. On physical examination, there is a firm, nontender, 10-cm mass in the left lateral posterior neck that appears to be fused cervical lymph nodes. Hepatosplenomegaly is noted. A head CT scan reveals a mass in the Waldeyer ring near the pharynx. A biopsy of the neck mass is performed, and on microscopic examination shows abnormal lymphoid cells with many mitotic figures and many apoptotic nuclei. He is treated with a cocktail of cell cycle-acting chemotherapeutic agents. The cervical and oral masses shrink dramatically over the next month. Based on his history and response to treatment, the tumor cells are most likely to have which of the following features? A Diminished vascularity B Evolution of polyclonality C High growth fraction D Limited capacity to metastasize E Strong expression of tumor antigens

*The answer is D.* Polycyclic aromatic hydrocarbons, originally derived from coal tar, are among the most extensively studied carcinogens. These compounds produce cancers at the site of application. Since polycyclic hydrocarbons have been identified in cigarette smoke, it has been suggested (but not proved) that they are involved in the pathogenesis of lung cancer. Aflatoxin B1 (choice A), a natural product of the fungus Aspergillus flavus, is among the most potent liver carcinogens. Asbestos (choice B), a mineral, is associated with mesothelioma and adenocarcinoma of lung. Industrial workers exposed to high levels of vinyl chloride (choice E) in the ambient atmosphere developed angiosarcomas of the liver. *Diagnosis:* Squamous cell carcinoma of lung

A 60-year-old man presents with a 4-month history of increasing weight loss, wheezing, and shortness of breath. He has smoked two packs of cigarettes a day for 40 years. His past medical history is significant for emphysema and chronic bronchitis. A chest X-ray shows a 10-cm mass in the left lung. Bronchoscopy discloses obstruction of the left main stem bronchus. A biopsy is obtained (shown in the image). Which of the following potent carcinogens was most likely involved in the pathogenesis of lung cancer in the patient described? (A) Aflatoxin B1 (B) Asbestos (C) Azo dyes (D) Polycyclic aromatic hydrocarbons (E) Vinyl chloride

*The answer is D.* Tumor markers are products of malignant neoplasms that can be detected in cells or body fluids. Useful tumor markers include immunoglobulins, fetal proteins, enzymes, hormones, and cytoskeletal proteins. Carcinomas uniformly express cytokeratins, which are intermediate filaments. Alpha-fetoprotein (choice A) is a marker for yolk sac carcinoma and hepatocellular carcinoma. Calretinin (choice B) provides a marker for mesothelioma. Carcinoembryonic antigen (choice C) is a marker for colon carcinoma and many other malignancies. Synaptophysin (choice E) is a marker for neuroendocrine tumors, including small cell carcinoma of the lung. *Diagnosis:* Squamous cell carcinoma of lung

A 60-year-old man presents with a 4-month history of increasing weight loss, wheezing, and shortness of breath. He hasmsmoked two packs of cigarettes a day for 40 years. His past medical history is significant for emphysema and chronic bronchitis. A chest X-ray shows a 10-cm mass in the left lung. Bronchoscopy discloses obstruction of the left main stem bronchus. A biopsy is obtained (shownn in the image). Immunohistochemical studies of this biopsy specimen would most likely show strong expression of which of the following tumor markers? (A) Alpha-fetoprotein (B) Calretinin (C) Carcinoembryonic antigen (D) Cytokeratins (E) Synaptophysin

*The answer is A.* Transitional cell carcinoma is the most common malignant tumor of the urinary bladder, and the incidence of bladder cancer is increased in aniline dye workers. These azo dyes are converted to water-soluble carcinogens in the liver. They are excreted in the urine, where they primarily affect the transitional epithelium of the bladder. Benzene exposure (choice C) is associated with leukemia. Vinyl chloride exposure (choice E) has been associated with hepatic angiosarcomas. *Diagnosis:* Transitional cell carcinoma of bladder

A 60-year-old man who worked for 30 years in a chemical factory complains of blood in his urine. Urine cytology discloses dysplastic cells. A bladder biopsy demonstrates transitional cell carcinoma. Which of the following carcinogens was most likely involved in the pathogenesis of bladder cancer in this patient? (A) Aniline dyes (B) Arsenic (C) Benzene (D) Cisplatinum (E) Vinyl chloride

*The answer is A.* Some chronic hepatitis B and C viral infections progress to hepatocellular and/or cholangiolar carcinoma. α-Fetoprotein (AFP) is an oncofetal protein that is a tumor marker for hepatocellular carcinomas and some testicular carcinomas. AFP is normally present in fetal life but not in adults. A serum immunoglobulin level with protein electrophoresis aids in the diagnosis of myeloma. Gastrointestinal tract adenocarcinomas, including those arising in the stomach, colon, and pancreas, as well as some lung carcinomas, may be accompanied by elevations in the serum carcinoembryonic antigen level. CA-19-9 is a tumor marker for colonic and pancreatic carcinomas. Some thyroid carcinomas produce calcitonin. Unfortunately, the sensitivity and specificity of tumor marker tests for detection of cancers, when they are small, is not high.

A 61-year-old man with a history of chronic viral hepatitis has noted a 6-kg weight loss over the past 5 months. Physical examination shows no masses or palpable lymphadenopathy. An abdominal CT scan shows a nodular liver with a 10-cm mass in the right lobe. A stool guaiac test result is negative. An elevation in which of the following laboratory tests is most likely to be present in this man? A Alpha-fetoprotein B CA-19-9 C Calcitonin D Carcinoembryonic antigen E Immunoglobulin M

*The answer is E.* Although neoplasms begin as monoclonal proliferations, additional mutations occur over time, leading to subclones of neoplastic cells with various aggressive properties. This subcloning may allow metastases, greater invasiveness, resistance to chemotherapy, and morphologic differences to occur. Because sarcomas of the lung are rare, the lung mass is statistically a metastasis. Though second primary malignancies do arise, particularly in persons who have already had a malignancy, the odds favor a metastasis in a person with a prior malignancy. Sarcomas are not related to smoking tobacco. Inheritance of a mutant RB gene is most likely to lead to childhood retinoblastomas and osteosarcomas. Kaposi sarcoma is the sarcoma most often associated with AIDS from HIV infection.

A 61-year-old woman has noted a feeling of pelvic heaviness for the past 6 months. On physical examination, there is a palpable nontender lower abdominal mass. An abdominal ultrasound scan shows a 12-cm solid mass in the uterine wall. A total abdominal hysterectomy is performed. The mass has the microscopic appearance of a well-differentiated leiomyosarcoma. One year later, a chest radiograph shows a 4-cm nodule in her right lower lung. Cytologic analysis of a fine-needle biopsy specimen of the nodule shows a poorly differentiated sarcoma. The patient's medical history indicates that she has smoked cigarettes most of her adult life. Which of the following mechanisms best explains these findings? A Continued cigarette smoking by the patient B Development of a second primary neoplasm C Inheritance of a defective RB gene D Immunodeficiency with HIV infection E Metastasis from an aggressive tumor subclone

*The answer is B.* A large, irregular, ulcerated mass such as that described is most likely malignant, and the epithelium of the bladder gives rise to carcinomas. Urothelial carcinomas are associated with smoking. An adenoma is a benign epithelial neoplasm of glandular tissues. A fibroma is a benign mesenchymal neoplasm. A papilloma is a benign, localized mass that has an exophytic growth pattern. A sarcoma is derived from cells of mesenchymal origin; sarcomas are much less common than carcinomas.

A 62-year-old man has had several episodes of hematuria in the past week. He has a 48 pack-year history of smoking cigarettes. On physical examination, there are no abnormal findings. A urinalysis shows 4+ hematuria, and cytologic examination of the urine shows that atypical cells are present. A cystoscopy is performed and a 4-cm sessile mass with a nodular, ulcerated surface is seen in the dome of the bladder. Which of the following terms best describes this lesion? A Adenoma B Carcinoma C Fibroma D Papilloma E Sarcoma

*The answer is B.* Cadherins are Ca2+-dependent transmembrane glycoproteins that mediated cell-cell adhesion. E-cadherin is expressed on the surface of all epithelia and mediates cell adhesion by "zipper-like" interactions. Overall, cadherins suppress invasion and metastasis. Thus, it is perhaps not surprising that the expression of E-cadherin is reduced in most carcinomas. Desmin (choice A) is an intermediate filament protein found in cells of mesenchymal origin. Lysyl hydroxylase (choice C) is involved in the post-translational modification of collagen. P selectin is a cell adhesion molecule that mediates the margination of neutrophils during acute inflammation. Telomerase (choice E) is increased in certain malignancies. *Diagnosis:* Breast cancer

A 62-year-old woman presents with a breast lump that she discovered 6 days ago. A breast biopsy shows lobular carcinoma in situ. Compared to normal epithelial cells of the breast lobule, these malignant cells would most likely show decreased expression of which of the following proteins? (A) Desmin (B) E-cadherin (C) Lysyl hydroxylase (D) P selectin (E) Telomerase

*The answer is E.* p53 is the most common target for genetic alterations in human neoplasms. Most are sporadic mutations, although some are inherited. The inheritance of one faulty p53 suppressor gene predisposes to a "second hit" that eliminates the remaining p53 gene. Homozygous loss of the p53 genes dysregulates the repair of damaged DNA, predisposing individuals to multiple tumors, as in this case. The APC gene is mutated in sporadic colon cancers and in familial polyposis coli. The BCL2 gene is mutated in some non-Hodgkin lymphomas. The HER2 gene is one of the EGF receptor family members amplified in some breast cancers. The EGF mutation is most often seen in squamous cell carcinomas of the lung. K-RAS mutations are present in many cancers, but not typically in lymphoid malignancies. The NF1 gene mutation is seen in neurofibromatosis type 1.

A 63-year-old man has a cough with hemoptysis for 10 days. He has a 65 pack-year history of smoking. A chest CT scan shows a 5-cm right hilar mass. Bronchoscopy is performed, and lung biopsy specimens show small cell anaplastic lung carcinoma. His family history shows three first-degree maternal relatives who developed leukemia, sarcoma, and carcinoma before age 40 years. Which of the following gene products is most likely to have been altered by mutation to produce these findings? A APC (tumor suppressor) B BCL2 (anti-apoptosis) C K-RAS (GTP binding) D NF1 (GTPase activation) E p53 (DNA damage response)

*The answer is B.* Localized, disordered differentiation during embryonic development results in a hamartoma, a disorganized caricature of normal tissue components. Such tumors, which are not strictly neoplasms, contain varying combinations of cartilage, ducts or bronchi, connective tissue, blood vessels, and lymphoid tissue. Ectopic islands of normal tissue (choice C), called choristoma, may also be mistaken for true neoplasms. These small lesions are represented by pancreatic tissue in the wall of the stomach or intestine, adrenal rests under the renal capsule, and nodules of splenic tissue in the peritoneal cavity. *Diagnosis:* Hamartoma

A 63-year-old woman with chronic bronchitis presents with shortness of breath. A chest X-ray reveals a 2-cm "coin lesion" in the upper lobe of the left lung. A CT-guided lung biopsy is obtained. Which of the following describes the histologic features of this lesion if the diagnosis is hamartoma? (A) Benign neoplasm of epithelial origin (B) Disorganized normal tissue (C) Ectopic islands of normal tissue (D) Granulation tissue (E) Granulomatous inflammation

*The answer is B.* Cancers may produce remote effects, collectively termed paraneoplastic syndromes. For example, the secretion of corticotropin (ACTH) by a tumor leads to clinical features of Cushing syndrome, including hyperglycemia and hypertension. Corticotropin production is most commonly seen with cancers of the lung, particularly small cell carcinoma. Adrenal and pituitary metastases (choices A and D) would lead to loss of adrenal function (Addison disease). Although pituitary adenoma (choice C) is a possible cause of Cushing syndrome, this choice would be unlikely in a patient with lung cancer. *Diagnosis:* Small cell carcinoma of lung, paraneoplastic syndrome

A 65-year-old man complains of muscle weakness and a dry cough for 4 months. He has smoked two packs of cigarettes daily for 45 years. A chest X-ray shows a 4-cm central, left lung mass. Laboratory studies reveal hyperglycemia and hypertension. A transbronchial biopsy is diagnosed as small cell carcinoma. Metastases to the liver are detected by CT scan. Which of the following might account for the development of hyperglycemia and hypertension in this patient? (A) Adrenal metastases (B) Paraneoplastic syndrome (C) Pituitary adenoma (D) Pituitary metastases (E) Thrombosis of the renal artery

*The answer is D.* Angiogenesis is a requirement for the continued growth of cancers, whether primary or metastatic. In the absence of new vessels to supply the nutrients and remove waste products, malignant tumors do not grow larger than 1 to 2 mm in diameter. In general, causes of tumor cell death in situ include (1) programmed cell death (apoptosis); (2) inadequate blood supply, with consequent ischemia; (3) a paucity of nutrients; and (4) vulnerability to specifi c and nonspecific host defenses. *Diagnosis:* Metastatic cancer

A 65-year-old man dies after a protracted battle with metastatic colon carcinoma. At autopsy, the liver is filled with multiple nodules of cancer, many of which display central necrosis (umbilication). Which of the following best explains the pathogenesis of tumor umbilication in this patient? (A) Biphasic tumor (B) Chronic inflammation (C) Granulomatous inflammation (D) Ischemia and infarction (E) Stimulation of angiogenesis

*The answer is E.* Basal cell carcinoma (BCC) is the most common malignant tumor in persons with pale skin. BCC usually develops on the sun-damaged skin of people with fair skin and freckles. There is a direct correlation between total exposure to sunlight and the incidence of BCC, as well as squamous cell carcinoma and melanoma. The deleterious effects of sunlight (UV radiation) include enzyme inactivation, mutagenesis, and cell death. Divalent metal cations such as nickel, lead, cadmium, cobalt, and beryllium (choice B) can react with biomolecules and induce cancer. Most metal-induced cancers occur in an occupational setting; however the carcinogenic mechanisms are unknown. *Diagnosis:* Basal cell carcinoma

A 65-year-old man presents with a pearly papule on his upper lip (patient shown in the image). A biopsy reveals buds of atypical, deeply basophilic keratinocytes extending from the overlying epidermis into the papillary dermis. Which of the following carcinogenic stimuli was the most important risk factor for development of this patient's skin cancer? (A) Aflatoxin B1 (B) Divalent metal cations (C) Aromatic amines and azo dyes (D) Vinyl chloride (E) Sunlight

*The answer is C.* Carcinomas metastasize through lymphatics most often, usually to regional nodes first. Hematogenous metastases are possible, however, to sites such as bone marrow, liver, or the opposite lung. About half of all cerebral metastases arise from lung primary carcinomas. Soft-tissue metastases to muscle, fat, and connective tissues are rare, as are splenic metastases.

A 66-year-old man with chronic cough has an episode of hemoptysis. On physical examination, there are no abnormal findings. A chest radiograph shows a 6-cm mass in the right lung. A sputum cytologic analysis shows neoplastic squamous cells. Metastases from his lung lesion are most likely to be found at which of the following sites? A Cerebral hemisphere B Chest wall muscle C Hilar lymph nodes D Splenic red pulp E Vertebral bone marrow

*The answer is E.* Cancer cells often display loss of cell cycle restriction point control through mechanisms such as overexpression of cyclin D1, loss of Cdk inhibitors, or inactivation of the pRb or p53 proteins. The p53 gene is deleted or mutated in 75% of cases of colorectal cancer and frequently mutated in numerous other tumors. The p53 protein is a negative regulator of cell division. Inactivating mutations of p53 cause loss of cell cycle restriction point control and allow cells with damaged DNA to progress through the cell cycle. Malignant cells have increased cellular motility (see choice A), reduced stem cell differentiation (see choice B), decreased cell adhesion (see choice C), and decreased susceptibility to apoptosis (see choice D). *Diagnosis:* Adenocarcinoma of colon

A 68-year-old man complains of recent changes in bowel habits and blood-tinged stools. Colonoscopy reveals a 3-cm mass in the sigmoid colon. Biopsy of the mass shows infiltrating malignant glands. These neoplastic cells have most likely acquired a set of mutations that cause which of the following changes in cell behavior? (A) Decreased cellular motility (B) Enhanced stem cell differentiation (C) Increased cell-cell adhesion (D) Increased susceptibility to apoptosis (E) Loss of cell cycle restriction point control

*The answer is B.* The characteristic tumor associated with asbestos exposure is mesothelioma of the pleural and peritoneal cavities. This cancer has been reported to occur in 2% to 3% of heavily exposed workers. The pipe fitters in shipyards were the most exposed workers. Many of these workers developed mesotheliomas 20 to 40 years after exposure. It is reasonable to surmise that mesotheliomas of both the pleura and the peritoneum reflect the close contact of these membranes with asbestos fibers transported to them by lymphatic channels. Like the polycyclic aromatic hydrocarbons, aflatoxin B1 (choice A) can bind covalently to DNA and is among the most potent liver carcinogens recognized. Beryllium (choice C) and silica (choice E) cause lung disease, but they are not carcinogenic. *Diagnosis:* Mesothelioma

A 68-year-old man who has worked in a shipyard and manufacturing plant all his adult life complains of a 4-month history of chest discomfort, malaise, fever, night sweats, and weight loss. A chest X-ray reveals a large pleural effusion. The patient dies 5 months later of cardiorespiratory failure. The lung at autopsy is shown in the image. This malignant neoplasm is associated with environmental exposure to which of the following carcinogens? (A) Aflatoxin B1 (B) Asbestos (C) Beryllium (D) Ionizing radiation (E) Silica

*The answer is D.* The RAS oncogene is the most common oncogene involved in the development of human cancers. Mutations of the RAS oncogene reduce GTPase activity, and RAS is trapped in an activated GTP-bound state. RAS then signals the nucleus through cytoplasmic kinases. The APC gene can cause activation of the WNT signaling pathway. The MYC oncogene is a transcriptional activator that is overexpressed in many tumors. The p53 tumor suppressor gene encodes a protein involved in cell cycle control. The RET proto-oncogene encodes a receptor tyrosine kinase involved in neuroendocrine cells of the thyroid, adrenal medulla, and parathyroids. The sis oncogene encodes platelet-derived growth factor receptor-β, which is overexpressed in certain astrocytomas.

A 70-year-old woman reported a 4-month history of a 4-kg weight loss and increasing generalized icterus. On physical examination, she has midepigastric tenderness on palpation. An abdominal CT scan shows a 5-cm mass in the head of the pancreas. Fine-needle aspiration of the mass is performed. On biochemical analysis, the neoplastic cells show continued activation of cytoplasmic kinases. Which of the following genes is most likely to be involved in this process? A APC B MYC C p53 D RAS E RET F sis

*The answer is E.* TGF-β inhibits cell proliferation by activation of growth-inhibiting genes, such as the CDKIs. All pancreatic cancers and 83% of colon cancers have at least one mutational event in a TGF-β pathway. The BCL2 family of genes acts as a regulator of apoptosis. The β-catenin pathway seen with the APC gene is involved with growth regulation; loss of the APC gene loci leads to failure in destruction of β-catenin, which translocates to the cell nucleus, where it functions as a transcription factor promoting growth. The MYC gene is a target of the activated RAS pathway. The p53 protein is involved in tumor suppression.

A 76-year-old man has experienced abdominal pain for the past year. On physical examination, there is an epigastric mass. An abdominal CT scan shows a 10-cm mass in the body of the pancreas. A fine-needle biopsy specimen of this mass shows a moderately differentiated adenocarcinoma. Mutational analysis of the carcinoma cells shows inactivation of cyclin-dependent kinase inhibitor with loss of growth-suppression. Regulatory pathways controlled by which of the following genes are most likely altered in this man's carcinoma? A BCL2 B β-Catenin C MYC D p53 E TGF-β

*The answer is F.* In the figure the disorderly, atypical epithelial cells involve the entire thickness of the epithelium. They extend through the underlying basement membrane and into the underlying stroma as rounded nests at the right, a process known as invasion. The ectocervix and the squamous metaplasia of endocervix give rise to dysplasia from which squamous cell carcinoma can arise. Carcinoma in situ is confined to the epithelium; if the basement membrane is breached, the lesion is no longer in situ, but rather invasive. An adenocarcinoma is a malignant neoplasm arising from glandular epithelium, such as the endocervix or endometrium, not the ectocervix. A dysplastic process could precede development of carcinoma in situ and squamous carcinoma; dysplasia involves only part of the thickness of the epithelium. A hamartoma contains a mixture of cell types common to a tissue site. Metaplasia can occur in response to persistent infection with human papillomavirus (HPV) and other inflammatory conditions. Metaplasia can be the precursor to dysplasia.

A Pap smear obtained from a 29-year-old woman during a routine health maintenance examination is abnormal. She is currently asymptomatic. She has a history of multiple sexual partners. Cervical biopsy specimens are obtained and the microscopic appearance is shown in the figure. Which of the following is the most likely diagnosis? A Adenocarcinoma B Carcinoma in situ C Dysplasia D Hamartoma E Metaplastic transformation F Squamous cell carcinoma

*The answer is B.* Cytokeratin is an epithelial cell marker.

A pathologist uses monoclonal antibodies against several intermediate filament proteins and finds that a tumor section stains positive for cytokeratin only. The tumor most likely originated from which of the following tissues? (A) Connective (B) Epithelial (C) Glial (D) Muscle (E) Nemal

*The answer is C.* Four DNA viruses (human papillomavirus, Epstein-Barr virus [EBV], hepatitis B virus, and herpesvirus-8) are incriminated in the development of human cancers. EBV was the first virus to be unequivocally linked to the development of a human tumor. In 1958, Burkitt described a form of childhood lymphoma in a geographical belt across equatorial Africa, which he suggested might have a viral etiology. A few years later, Epstein and Barr discovered viral particles in cell lines cultured from patients with Burkitt lymphoma. African Burkitt lymphoma is a B-cell tumor, in which the neoplastic lymphocytes invariably contain EBV in their DNA and manifest EBV-related antigens. EBV does not infect the other choices. *Diagnosis:* Burkitt lymphoma, EBV

An 8-year-old African boy presents with swelling in his jaw and massive facial disfiguration. Biopsy reveals a tumor invading the bone marrow of the jaw. The pathogenesis of this malignant neoplasm is associated with a virus that exhibits a tropism for which of the following cells? (A) Chondrocytes (B) Fibroblasts (C) Lymphocytes (D) Macrophages (E) Osteocytes

*The answer is E.* Xeroderma pigmentosum is an autosomal recessive disease in which increased sensitivity to sunlight is accompanied by a high incidence of skin cancers, including basal cell carcinoma, squamous cell carcinoma, and malignant melanoma. Several xeroderma pigmentosum genes are involved in nucleotide excision of ultraviolet-damaged DNA. Li-Fraumeni syndrome (choice C) refers to an inherited predisposition to develop cancers in many organs due to germline mutations of p53. Ataxia telangiectasia (choice A) features cerebellar degeneration, immunologic abnormalities, and a predisposition to cancer. The mutated gene codes for a nuclear phosphoprotein involved in regulation of the cell cycle and DNA repair. Patients with hereditary albinism (choice B) are also at high risk for development of squamous cell carcinoma of the skin, but they do not have a defect in DNA excision repair. Patients with neurofibromatosis (choice D) develop benign cutaneous neurofibromas. *Diagnosis:* Xeroderma pigmentosum

An 8-year-old girl with numerous hypopigmented, ulcerated, and crusted patches on her face and forearms develops an indurated, crater-like, skin nodule on the back of her left hand. Biopsy of this skin nodule discloses a squamous cell carcinoma. Molecular biology studies reveal that this patient has germline mutations in the gene encoding a nucleotide excision repair enzyme. What is the appropriate diagnosis? (A) Ataxia telangiectasia (B) Hereditary albinism (C) Li-Fraumeni syndrome (D) Neurofi bromatosis, type I (E) Xeroderma pigmentosum

*The answer is D.* To establish criteria for therapy, many cancers are classified according to histologic grading schemes or by staging protocols that describe the extent of spread. Cancer grading reflects cellular characteristics. Low-grade tumors are well differentiated, whereas high-grade tumors lack differentiated features (anaplasia). The general correlation between cytologic grade and the behavior of a neoplasm is not invariable. Indeed, there are many examples of tumors of low cytologic grades that exhibit substantial malignant properties. The other choices pertain to cancer staging. *Diagnosis:* Prostate cancer

An 80-year-old man complains of lower abdominal pain, increasing weakness, and fatigue. He has lost 16 lb (7.3 kg) in the past 6 months. The prostate-specific antigen test is elevated (8.5 ng/mL). Rectal examination reveals an enlarged and nodular prostate. A needle biopsy of the prostate discloses invasive prostatic adenocarcinoma. Histologic grading of this patient's carcinoma is based primarily on which of the following criteria? (A) Capsular involvement (B) Extent of regional lymph nodes involvement (C) Pulmonary metastases (D) Resemblance to normal tissue of origin (E) Volume of prostate involved by tumor

*The answer is E.* Development of colonic adenocarcinoma typically takes years, during which time multiple mutations occur within the mucosa, including mutations involving such genes as APC (adenomatous polyposis coli), K-RAS, and p53. The accumulation of mutations, rather than their occurrence in a specific order, is most important in the development of a carcinoma. Activation of proto-oncogenes, extensive regeneration, faulty TP53 genes, and amplification of growth factor receptor genes all contribute to the development of malignancies, but they are not sufficient by themselves alone to produce a carcinoma from an adenoma of the colon. Inherited loss of wild-type TP53 contributes to multiple cancers, but not to sporadic adenomas.

An epidemiologic study investigates the potential morphologicand molecular alterations that may contribute to the development of cancers in a population. Data analyzed from resected colonic lesions show changes that suggest the evolution of a sporadic colonic adenoma into an invasive carcinoma. Which of the following best describes the mechanism producing these changes leading to colonic malignancies? A Activation of proto-oncogenes by chromosomal translocation B Extensive regeneration of tissues increasing the mutation rate in regenerating cells C Inheritance of defects in TP53 genes that increase the susceptibility to develop cancer D Overexpression of growth factor receptor genes E Stepwise accumulation of multiple proto-oncogene and tumor suppressor gene mutations

*The answer is B.* Incidence of lung cancers increased dramatically in the 20th century because of the popularity of cigarette smoking. As the number of individuals in a population who smoke increases, so do the number of lung cancers. Some cancers of the urinary tract, oral cavity, esophagus, and pancreas also are causally related to smoking. Breast, prostate, and colon cancers remain common in developed nations, but the number of cases has not increased sharply. Pap smear screening and human papillomavirus (HPV) vaccination markedly decreases numbers of cervical cancers. There has been an increase in the incidence of melanomas worldwide, but there are still far fewer cases of melanomas than of lung cancers. Hepatic and intracranial neoplasms in adults are far less common than lung cancers.

An epidemiologic study of cancer deaths recorded in the last half of the 20th century is conducted. The number of deaths for one particular cancer had increased markedly in developed nations. More than 30% of cancer deaths in men, and more than 24% of cancer deaths in women, were caused by this neoplasm in 1998. In some nations, prevention strategies reduced deaths from this cancer. Which of the following neoplasms was most likely identified by this study? A Cerebral glioma B Bronchogenic carcinoma C Hepatocellular carcinoma D Colonic adenocarcinoma E Pancreatic adenocarcinoma F Skin melanoma

*The answer is B.* The decrease in the number of gastric cancers may be related to reduced numbers of dietary carcinogens or a decrease in the prevalence of Helicobacter pylori infection; however, the exact reason is obscure. Cerebral gliomas are not as common as carcinomas; an urban legend links them to cell phone use, but legitimate epidemiologic studies have not made this link. Angiosarcomas of the liver are quite rare; they are epidemiologically linked to vinyl chloride exposure. Leukemias and lymphomas are not as common as carcinomas. Pulmonary small cell carcinomas are related to smoking, and the numbers have decreased in many countries with campaigns to reduce smoking; the death rate is typically high because the prognosis for lung cancer is so poor.

An epidemiologic study of cancer deaths recorded in the last half of the 20th century is conducted. The number of deaths for one particular type of cancer had been decreasing in developed nations, despite the absence of widespread screening and prevention programs. Which of the following neoplasms was most likely to be identified by this study? A Cerebral glioma B Gastric adenocarcinoma C Hepatic angiosarcoma D Leukemia E Pulmonary small cell carcinoma

*The answer is B.* Patients with hereditary nonpolyposis colon carcinoma (HNPCC) inherit one defective copy of mismatch repair genes. Several human mismatch repair genes are involved in the development of HNPCC. Mismatch repair defects have microsatellite instability. Microsatellites are tandem repeats found throughout the genome. Normally, the length of these microsatellites remains constant. In HNPCC, these satellites are unstable and increase or decrease in length. Although HNPCC accounts for only 2% to 4% of all colon carcinomas, microsatellite instability can also be detected in about 15% of all sporadic colon carcinomas. Mutations in mismatch repair genes can be detected by the presence of microsatellite instability. The other listed options are not characteristic of HNPCC.

An investigational study reviews cells harvested from patients 30 to 50 years of age who had right-sided colon cancer with multiple polyps present. These patients typically develop multiple malignant lesions of the colon during middle age. Molecular analysis of the cells from the lesions shows changes in hPMS1, hPMS2, and hMLH1 genes. Which of the following principles of carcinogenesis is best illustrated by this study? A Carcinogenesis is a multistep process B Inability to repair DNA is carcinogenic C Many oncogenes are activated by translocations D Tumor initiators are mutagenic E Tumor promoters induce proliferation

*The answer is A.* Many human cancers show abnormalities in the control of apoptosis. For example, follicular B-cell lymphomas display a characteristic chromosomal translocation in which the bcl-2 gene is brought under the transcriptional control of the immunoglobulin light-chain gene promoter, thereby causing overexpression of bcl-2. As a result of the antiapoptotic properties of bcl-2, the neoplastic clone accumulates in lymph nodes. Since its demonstration in follicular lymphomas, bcl-2 expression has been observed in a variety of other human cancers. None of the other choices describes the function of bcl-2. *Diagnosis:* Follicular lymphoma

Cytogenetic studies in a 40-year-old woman with follicular lymphoma demonstrate a t(14;18) chromosomal translocation involving the bcl-2 gene. Constitutive expression of the protein encoded by the bcl-2 gene inhibits which of the following processes in this patient's transformed lymphocytes? (A) Apoptosis (B) DNA excision repair (C) G1-to-S cell cycle progression (D) Oxidative phosphorylation (E) Protein (N-linked) glycosylation

*The answer is E.* The best-known example of an acquired chromosomal translocation in a human cancer is the Philadelphia chromosome, which is found in 95% of patients with CML. The c-abl protooncogene on chromosome 9 is translocated to chromosome 22, it is placed in juxtaposition to the breakpoint cluster region (bcr). The c-abl gene and bcr region unite to produce a hybrid oncogene that codes for an aberrant protein with very high levels of tyrosine kinase activity, which generates mitogenic and antiapoptotic signals. *Diagnosis:* Chronic myelogenous leukemia, Philadelphia chromosome

Cytogenetic studies in a 70-year-old woman with chronic myelogenous leukemia (CML) demonstrate a t(9;22) chromosomal translocation. Which of the following best explains the role of this translocation in the pathogenesis of leukemia in this patient? (A) Altered DNA methylation status (B) Enhanced expression of telomerase gene (C) Expansion of a trinucleotide repeat (D) Inactivation of tumor suppressor protein (E) Protooncogene activation

*The answer is B.* The choice of surgical approach or treatment modalities is influenced more by the stage of a cancer than by its cytologic grade. The significant criteria used for staging vary with different organs. Commonly used criteria include (1) tumor size, (2) extent of local growth, (3) presence of lymph node metastases, and (4) presence of distant metastases. The other choices reflect grade of the tumor. *Diagnosis:* Renal cell carcinoma

During a routine checkup, a 50-year-old man is found to have blood in his urine. He is otherwise in excellent health. An abdominal CT scan reveals a 2-cm right renal mass. You inform the patient that staging of this tumor is key to selecting treatment and evaluating prognosis. Which of the following is the most important staging factor for this patient? (A) Histologic grade of the tumor (B) Metastases to regional lymph nodes (C) Proliferative capacity of the tumor cells (D) Somatic mutations in the p53 tumor suppressor gene (E) Tumor cell karyotype (aneuploidy)

*The answer is A.* All human nucleated cells express MHC class I antigens. CD8+ T cells recognize peptides presented by MHC class I antigens. In many tumors, especially melanomas, the tumor cells produce peptides that can be presented by MHC class I molecules. Such tumor-specific peptides are not produced by other cells, so the CD8+ T cells specific for such peptides lyse melanoma cells, but not normal melanocytes or other normal cells.

In a clinical trial, patients diagnosed with malignant melanoma are treated by infusion of autologous CD8+ T cells grown in vitro. These CD8+ T cells are known to kill melanoma cells, but not normal cells. Which of the following target antigens in the tumor cells are most likely recognized by these CD8+ T cells? A Class I MHC molecules with a melanoma cell peptide B Class I MHC molecules with a peptide from normal melanocytes and melanoma cells C Class I MHC molecules plus a peptide derived from carcinoembryonic antigen D Class II MHC molecules with a melanoma cell peptide E Class II MHC molecules with a peptide from normal melanocytes and melanoma cells F Class II MHC molecules with laminin receptors on melanoma cells

*The answer is B.* MicroRNAs (miRNAs) are encoded by about 5% of the human genome. miRNAs do not encode for proteins, but bind and inactivate or cleave mRNA, preventing translation of proteins by mRNA. This effectively silences gene expression without affecting the gene directly. There is abundant transfer RNA (tRNA) present in the cytoplasm that is not a rate-limiting step to translation. DNA methylation, particularly at CG dinucleotides, is a way of suppressing gene expression directly, as is seen with genomic imprinting. Mutations that occur in genes in DNA may result in reduced mRNA production or abnormal protein production, but mRNA itself is not mutated. Mitochondrial DNA (mtDNA) encodes for proteins that are mainly involved in oxidative phosphorylation metabolic pathways

In a study of patients with non-Hodgkin B cell lymphoma, a nuclear gene is found to be actively transcribed to mRNA and is transported into the cell cytoplasm. A protein is translated from this mRNA, with up-regulation of BCL2. In a control group without lymphoma, translation of the mRNA does not occur. How is the silencing of this active gene's mRNA most likely to occur? A Absence of tRNA B Binding to miRNA C Methylation of DNA D Mutation of mRNA E Up-regulation of mtDNA

*The answer is E.* Chromosomal telomere shortening in normal human cells limits their replicative potential and gives rise to replicative senescence. This occurs because most somatic cells lack the enzyme telomerase. Normal human stem cells do express telomerase. By contrast, 90% or more of human tumor cells show activation of telomerase, explaining continued tumor growth in the body and "immortalized" cell lines in culture. All other pathways listed cannot affect telomerase shortening, which is the rate-limiting step in indefinite replication of cells.

In an experiment, cells from human malignant neoplasms explanted into tissue culture medium continue to replicate. This allows development of "immortal" tumor cell lines that are extremely useful for the study of tumor biology and responses to therapeutic modalities. Activation of which of the following molecular components is most likely to endow these tumor cells with limitless replicative ability in vivo and in vitro? A Hypoxia-induced factor 1 B BCL2 gene C Cyclin-dependent kinase gene methylation D DNA replication repair E Telomerase

*The answer is A.* Hypermethylation of many tumor suppressor and DNA repair genes has been demonstrated in human tumors. The pathways controlled by these genes are, therefore, suppressed. For example, the normal p53 gene can be inactivated by hypermethylation. Thus, aberrant methylation of tumor suppressor genes may be an epigenetic mechanism for a "second hit," leading to loss of heterozygosity. Unlike genetic changes in cancer, epigenetic changes are reversible, and a search for drugs that influence DNA methylation is under way. The other choices are unrelated to DNA methylation.

Laboratory studies of a surgical specimen obtained from a patient described demonstrate hypermethylation of the p53 gene. Which of the following best characterizes this biochemical change in the neoplastic cells? (A) Epigenetic modification (B) Gene amplification (C) Insertional mutagenesis (D) Nonreciprocal translocation (E) Protooncogene mutation

*The answer is E.* Chromosomal alterations that result in an increased number of copies of a gene have been found primarily in solid tumors. Such aberrations are recognized as (1) homogeneous staining regions (HSRs); (2) abnormal banding regions on chromosomes; or (3) double minutes, which are visualized as small, paired cytoplasmic bodies. In some cases, gene amplification has been shown to involve protooncogenes. For example, HSRs may be seen in neuroblastomas and are all derived from the N-myc protooncogene. The presence of N-myc HSRs is associated with up to 700-fold amplifi cation of this gene and is a marker of advanced disease with a poor prognosis. Although the other choices are mechanisms for proto-oncogene activation, they do not cause upregulation of N-myc in patients with neuroblastoma. *Diagnosis:* Neuroblastoma

The parents of a 6-month-old girl palpate a mass on the left side of the child's abdomen. Urinalysis shows high levels of vanillylmandelic acid. A CT scan reveals an abdominal tumor and bony metastases. The primary tumor is surgically resected. Histologic examination of the surgical specimen discloses neuroblastoma. Evaluation of the N-myc protooncogene in this child's tumor will most likely demonstrate which of the following genetic changes? (A) Chromosomal translocation (B) Exon deletion (C) Expansion of a trinucleotide repeat (D) Frameshift mutation (E) Gene amplification

*The answer is B.* More intermediate substances, such as lipids and amino acids, needed for cancer cell growth are generated by aerobic glycolysis than by oxidative phosphorylation.

Which of the following statement is true regarding the respiratory metabolism of cancer cells? A. More ATPs are generated by cancer cell using aerobic glycolysis than by oxidative phosphorylation B. More intermediate substances, such as lipids and amino acids, needed for cancer cell growth are generated by aerobic glycolysis than by oxidative phosphorylation C. The same number of ATPs are generated by cancer cells using either aerobic glycolysis or oxidative phosphorylation D. The Warburg effect seen in cancer cells refers to exclusive use of oxidative phosphorylation E. Aerobic glycolysis requires IDH1 (isocitrate dehydrogenase) mutation.


Conjuntos de estudio relacionados

History of Rock N Roll People and Places

View Set

CHM2046L Final Exam UF Spring 2021

View Set

Business Law - Final (Chapter 3, 5, 7, 8, 9, & 50)

View Set

AGEC 1003 Fannin Final Exam Review Questions

View Set